2012 Baltic Way 팀수학경시대회 3번문제

a) 방정식 \[ \lfloor x\rfloor (x^2+1)=x^3\]을 만족하는 실수해는 연속한 두 양의 정수 사이의 구간에는 정확히 하나밖에 없음을 보여라. ($\lfloor x\rfloor$는 $x$보다 크지 않은 정수 중 제일 큰 것을 뜻한다.)
b) 모든 양의 실수해는 유리수가 아님을 보여라.

GD Star Rating
loading...